Question

Solve the problem. Let H be the set of all polynomials having degree at most 4 and rational coefficients. Determine whether H

0 0
Add a comment Improve this question Transcribed image text
Answer #1

Solution:

Given that

H=\left \{ a+bx+cx^{2}+dx^{3}+ex^{4}:a,b,c,d,e \,\varepsilon Q \right \}

(a) H contains Zero vectors.

(b)

H is closed under vector addition

\therefore Sum of two rational no is again a a rational number.

Let,

P_{1}(x)=a_{1}+b_{1}x+c_{1}x^{2}+d_{1}x^{3}+e_{1}x^{4}

P_{2}(x)=a_{2}+b_{2}x+c_{2}x^{2}+d_{2}x^{3}+e_{2}x^{4}

P_{1}(x)+P_{2}(x)=(a_{1}+a_{2})+(b_{1}+b_{2})x+(c_{1}+c_{2})x^{2}+(d_{1}+d_{2})x^{3}+(e_{1}+e_{2})x^{4}\Rightarrow P_{1}(x)+P_{2}(x)\epsilon H

(c)

H is not closed under multiplication by scalars.

Let,

P(x)=1+2x+\frac{3}{2}x^{2}+\frac{1}{2}x^{3}+5x^{4}

Let  \alpha =\sqrt{2}

\alpha P(x)=\sqrt{2}+2\sqrt{2}x+\frac{3}{\sqrt{2}}x^{2}+\frac{1}{\sqrt{2}}x^{3}+5\sqrt{2}x^{4}

all the coefficients does not belong to Q.

\Rightarrow It is not closed under scalar multiplication.

Add a comment
Know the answer?
Add Answer to:
Solve the problem. Let H be the set of all polynomials having degree at most 4...
Your Answer:

Post as a guest

Your Name:

What's your source?

Earn Coins

Coins can be redeemed for fabulous gifts.

Not the answer you're looking for? Ask your own homework help question. Our experts will answer your question WITHIN MINUTES for Free.
Similar Homework Help Questions
  • Solve the problem. Let H be the set of all polynomials having degree at most 4...

    Solve the problem. Let H be the set of all polynomials having degree at most 4 and rational coefficients. Determine whether His a vector space. If it is not a vector space, determine which of the following properties it fails to satisfy A: Contains zero vector B: Closed under vector addition C Closed under multiplication by scala His not a vector space, not dosed under multiplication to scalars His a vector space His not a vector space:not closed under vector...

  • Let H be the set of third degree polynomials Let H be the set of third...

    Let H be the set of third degree polynomials Let H be the set of third degree polynomials {ax + ax? + ax3 | DEC} Is H a subspace of P3? Why or why not? Select all correct answer choices (there may be more than one). a. H is not a subspace of P3 because it is not closed under scalar multiplication b.H is a subspace of P3 because it contains the zero vector of P3 c. H is not...

  • Let    be the set of third degree polynomials Is    a subspace of   ? Why or why not?...

    Let    be the set of third degree polynomials Is    a subspace of   ? Why or why not? Select all correct answer choices (there may be more than one). a. is not a subspace of    because it is not closed under vector addition b. is a subspace of    because it contains the zero vector of   c. is not a subspace of    because it is not closed under scalar multiplication d. is a subspace of    because it contains only second degree polynomials e. is...

  • Let H be the set of third degree polynomials H = {ax + ax? + ax?...

    Let H be the set of third degree polynomials H = {ax + ax? + ax? | aEC} Is H a subspace of P3? Why or why not? Select all correct answer choices (there may be more than one). a. H is a subspace of P3 because it contains the zero vector of P3 b. H is a subspace of P3 because it is closed under vector addition and scalar multiplication Oc H is a subspace of P3 because it...

  • Let H be the set of third degree polynomials H = {ax + ax2 + ax...

    Let H be the set of third degree polynomials H = {ax + ax2 + ax aEC} Is H a subspace of P3? Why or why not? Select all correct answer choices (there may be more than one). 0 a. H is a subspace of P3 because it contains only second degree polynomials 1b. H is a subspace of P3 because it can be written as the span of a subset of P3 OcH is not a subspace of P3...

  • Let H be the set of third degree polynomials H = {ax + ax? + ax?...

    Let H be the set of third degree polynomials H = {ax + ax? + ax? | aEC} is H a subspace of P3? Why or why not? Select all correct answer choices (there may be more than one). a. H is a subspace of P3 because it contains the zero vector of P3 b.H is a subspace of P3 because it is closed under vector addition and scalar multiplication c. H is a subspace of P3 because it can...

  • Let H be the set of third degree polynomials H = {ax + ax? + ax3...

    Let H be the set of third degree polynomials H = {ax + ax? + ax3 | DEC} Is H a subspace of P3? Why or why not? Select all correct answer choices (there may be more than one). a. A is a subspace of P3 because it contains the zero vector of P3 | b. H is not a subspace of P3 because it does not contain the zero vector of P3 c. H is not a subspace of...

  • Determine if the given set is a subspace of P4. Justify your answer. All polynomials of...

    Determine if the given set is a subspace of P4. Justify your answer. All polynomials of degree at most 4, with integers as coefficients. Complete each statement below. The zero vector of P4 in the set because zero an integer The set v closed under vector addition because the sum of two integers an integer The set closed under multiplication by scalars because the product of a scalar and an integer an integer Is the set a subspace of P4?...

  • linear algebra 2 part mcq part a part b H Let be the set of third...

    linear algebra 2 part mcq part a part b H Let be the set of third degree polynomials H = {ax + ax? + ax' | AEC) P3 why or why not? H Is a subspace of Select all correct answer choices (there may be more than one). a. H P3 is a subspace of because it can be written as the span of a subset of b. H is a subspace of because it contains only second degree polynomials...

  • Determine if the set V = {at? | a € R} is a subspace of the...

    Determine if the set V = {at? | a € R} is a subspace of the vector space P2 = {ao +ajt + azt? | ao, a1, az ER}. You may assume that vector addition in P2 is given by the usual addition of polynomials and that the scalars used in scalar multiplication are real numbers. If you decide that Vis a subspace of P2, then identify the zero vector in V and explain briefly why Vis closed under vector...

ADVERTISEMENT
Free Homework Help App
Download From Google Play
Scan Your Homework
to Get Instant Free Answers
Need Online Homework Help?
Ask a Question
Get Answers For Free
Most questions answered within 3 hours.
ADVERTISEMENT
ADVERTISEMENT
ADVERTISEMENT